LSAT and Law School Admissions Forum

Get expert LSAT preparation and law school admissions advice from PowerScore Test Preparation.

 Robert Carroll
PowerScore Staff
  • PowerScore Staff
  • Posts: 1787
  • Joined: Dec 06, 2013
|
#105801
hifigirl,

Your diagram of the stimulus looks perfect!

Onto the answer choices:
In linking the contrapositive logic /ID --> /UR --> /EW, when investment is decreasing, the economy is NOT weak.
You diagrammed correctly but misspoke here - when investment is NOT decreasing, the economy is not weak. Your diagram proves that, and that's why answer choice (A) is correct.
UR --> ID but we know nothing else. Prices could remain constant as they are not linked to ID or UR.
All perfectly correct.
(C) MBT for same reasoning in (A)
Agreed, that's inferable by a valid chain.
EW --> PRC + UR allows for the economy to be weak AND prices to remain constant, if unemployment rate also decreases.
I think more should be said about this. The economy is definitely not weak. That's provable by the contrapositive of the chain. However, since that doesn't require prices to remain constant or otherwise, the second half of this answer could be true, making the entire "or" statement possible.
/PRC or /UR --> /EW allows for unemployment to rise AND economy to be not weak (if prices don't remain constant). It would hinge on activating the OR statement, suggesting /PRC --> /EW while unemployment rises.
We can more simply say that this answer must be true. The second element is definitely true, so the whole "or" is true.

Robert Carroll

Get the most out of your LSAT Prep Plus subscription.

Analyze and track your performance with our Testing and Analytics Package.